Small oscillations and spatial transformations | Part 1

In summary, the conversation discusses a transformation of spatial coordinates and its effects on potential and kinetic energies. It is stated that the transformed quantities will be indicated by ##'##. The conversation goes on to explain the conditions and equations involved in the transformation and finally, the question of how to prove that a certain statement is true is addressed. The expert summarizer provides a detailed summary of the conversation, highlighting key points and equations.
  • #1
JD_PM
1,131
158
TL;DR Summary
I am studying the theory of small oscillation (section 8 in the PDF I attached) and I do not fully understand it. That's why I want to discuss it in several parts.
Please note that the transformed quantities will be indicated by ##'##.

Let me give some context first.

Let us assume here that the general approximate form of the potential energy ##V## and the kinetic energy ##T## are given to be

$$V^{app} = q^T V q \tag 1$$

$$T^{app} = \dot q^T V \dot q \tag 2$$

We assume that energies are unaffected by spatial transformations. Such a condition yields the following equations

$$q^T V q = V^{app} =V'^{app} = q'^T V' q'\tag{3a}$$

$$\dot q^T T \dot q = T^{app} =T'^{app}= \dot q'^T T' \dot q'\tag{3b}$$

Let us work with the following transformation:

$$q' = Sq \tag4$$

Where ##S## is a squared orthogonal (i.e. ##S^T=S^{-1}##) matrix.

Let's plug ##(4)## into ##(3b)## to get that

$$T=S^T T' S \iff T'=(S^T)^{-1} T S^{-1} \tag5$$

Let us focus on ##T'##; As it equals to the diagonalizable condition (i.e. ##T'= STS^{-1}##), we can always find another transformation matrix that makes ##T'## a diagonal matrix (i.e. ##q' = S_1q## that makes ##T'=D##, where ##D## is an unknown diagonal matrix).

I understand everything so far.

Then the notes I am studying state that we can apply another transformation

$$q'' = S_2q'=S_1S_2q \tag6$$

Where they use ##q' = S_1q## instead of ##(4)##

But then they state that '##S_2=\sqrt{D}## and then ##T''= I##'

- How can I prove that the above sentence is indeed true?

- Why did they use ##q' = S_1q## instead of ##(4)##?

Any help is appreciated.

EDIT: note I asked this question https://math.stackexchange.com/questions/3741710/transformation-of-spatial-coordinates but got little attention. If you'd like to have more information please let me know.
 

Attachments

  • LectureNotes.CM.pdf
    183.5 KB · Views: 143
Last edited:
Physics news on Phys.org
  • #2
Hello,
before answering your question I would like to draw your attention on a sloppy statement of the PDF file you put in attachement. Right before eq. (40) it says that all square matrices are diagonalisable, which is obviously not true. Take ##\begin{pmatrix} 5 & 1 \\ 0 & 5 \end{pmatrix} ## for example, but in your case it doesn't matter since your are dealing with real symetric matrices which are diagonalisable by the spectral theorem.

For the question with ##S_{1}q##, this is just a labelling issue. The text use ##S## to show you general properties. Then to go more precise in the problem it is usual to give special label to the matrix you use, especially when there will be more than one. So in short ##S_{1}## is a matrix verifying the same properties as ##S## and ##P## in your particular case, and with an arbitrary label.

For the proof with ##S_{1}## and ##S_{2}##, you have ## T = S_{1}^{T}T'S_{1} =S_{1}^{T}D S_{1} ##, then if you define ## S_{2} = \sqrt{D}## you have ## S_{2}^{T} S_{2} = S_{2}^{T} I S_{2} = D## so by replacing ## D## in the previous expression by this one you have ## T = S_{1}D S_{1}= S_{1}^{T} S_{2}^{T} I S_{2} S_{1} = \left( S_{2} S_{1} \right) ^{T} T'' S_{2} S_{1} ## , which corresponds to the transformation ## q'' = S_{2} S_{1} ## and gives ##T'' = I##.
 
  • Like
Likes JD_PM
  • #3
Hi patfinder, thanks for your reply :smile:.

I've been working out this question so let's share and discuss.

patfinder said:
real symetric matrices which are diagonalisable by the spectral theorem.

Here I found the proof and others related to real symmetric matrices.

patfinder said:
For the proof with ##S_{1}## and ##S_{2}##, you have ## T = S_{1}^{T}T'S_{1} =S_{1}^{T}D S_{1} ##, then if you define ## S_{2} = \sqrt{D}## you have ## S_{2}^{T} S_{2} = S_{2}^{T} I S_{2} = D## so by replacing ## D## in the previous expression by this one you have ## T = S_{1}D S_{1}= S_{1}^{T} S_{2}^{T} I S_{2} S_{1} = \left( S_{2} S_{1} \right) ^{T} T'' S_{2} S_{1} ## , which corresponds to the transformation ## q'' = S_{2} S_{1} ## and gives ##T'' = I##.

I think we have done basically the same but let's see.

If we apply the coordinate transformation ##\vec q' = S_1 \vec q \ ## to ##T= \ \dot{\vec q'}^T T' \dot q '## we get

$$T=(S_1 \dot{\vec q})^T T'(S_1 \dot{\vec q}) = \dot{\vec q}^T S_1^T T'S_1 \dot{\vec q} \tag1$$

Thus we have

$$T = S_1^T T'S_1 \iff T'=S_1^{-T} T S_1^{-1} \tag2$$

As ##T'## is a real symmetric matrix, it is diagonalizable. Thus we can assert that

$$T' = D \tag3$$

Next if we apply the coordinate transformation ##\vec q'' = S_2 \vec q' \ ## to ##T'= \ \dot{\vec q''}^T T'' \dot q ''## we get

$$T'=(S_2 \dot{\vec q'})^T T''(S_2 \dot{\vec q'}) = \dot{\vec q'}^T S_2^T T''S_2 \dot{\vec q'} \tag4$$

Thus we have

$$T' = S_2^T T''S_2 \iff T''=S_2^{-T} T' S_1^{-1} \tag5$$

If we set ##S_2 = \sqrt{D}## then, out of ##(5)## and using ##(3)##, we get

$$T'' = \Big( \sqrt{D} \Big)^{-T} T'\Big( \sqrt{D} \Big)^{-1}=\Big( \sqrt{D} \Big)^{-T} D\Big( \sqrt{D} \Big)^{-1}=\Big( \sqrt{D} \Big)^{-1} D\Big( \sqrt{D} \Big)^{-1}=1 \tag6$$

Where I have used the fact that the inverse matrix of a non-singular symmetric matrix (in this case ##\Big( \sqrt{D} \Big)^{-1}##) is symmetric (i.e. ##\Big( \sqrt{D} \Big)^{-T}=\Big( \sqrt{D} \Big)^{-1}##).

Please let me know if you do not agree.
 
  • #4
Now I am stuck in two basic aspects:

1) To justify ##(3)## I said 'As ##T'## is a real symmetric matrix, it is diagonalizable'; OK but I'd like to work it out explicitly. My idea is this: as we know that ##T## is a real symmetric matrix, we know that ##T=D##. Thus:

$$T'= S_1^{-T} T S_1^{-1} = (S_1^{T})^{-1} D S_1^{-1} = (S_1^{-1})^{-1} D S_1^{-1} = S_1 D S_1^{-1} \tag7$$

Where I assumed that ##S_1## is an orthogonal matrix.

If we set ##S_1=D## we indeed get ##T'=D##. I am not sure if this method is OK.

2) Something interesting happens if I try to apply the trick I used at 1. to ##T''##

$$T'' = \Big( \sqrt{D}^{-1} \Big)^{T} T'\Big( \sqrt{D} \Big)^{-1}=\Big( \sqrt{D}^{-1} \Big)^{-1} T'\Big( \sqrt{D} \Big)^{-1}=\sqrt{D} D\Big( \sqrt{D} \Big)^{-1}=D \tag8$$

I get ##T'' = D## instead of ##T'' = 1##.

So making the assumption that ##(\sqrt{D})^{-1}## is orthogonal seems to be incorrect.

Thus there has to be something wrong in my reasoning... Thinking...
 
  • #5
Hi,
I think you are confused and mixing things up. First :
JD_PM said:
To justify (3) I said 'As ##T'## is a real symmetric matrix, it is diagonalizable'
There is no justification to be presented. ##T## is diagonalisable, saying ##T' = D## is just a matter of identification in my previous message.
Then I think you are trying to proove the spectral theorem with T. If you want to see how to do so there is plenty of proofs for the real symetric matrices case on the internet.

Then you are making a big mistake :

JD_PM said:
My idea is this: as we know that ##T## is a real symmetric matrix, we know that ##T = D## .
No ! ##T## is diagonalisable, simply gives you thaht there exist two matrices ## S_{1}## and ##D## with ##D## diagonal such that ##T = S_{1}^{-1} D S_{1}##. Here you are writing that ## T ## is diagonal straight away, which is wrong in general, and moreover you are mixing up the previous notations, as we simply identified ##D## and ##T'## before.

-Second:
JD_PM said:
If we apply the coordinate transformation to ##T= \ \dot{\vec q'}^T T' \dot q '## we get
Writing this should give you a shock ! You are wirting ##T## a matrix, ## = \ \dot{\vec q'}^T T' \dot q '## the kinetic energy, which is a scalar, a number ! So you are starting from a wrong relation.

The good transformation law is given in equation (39) in your pdf file.
To derive it, it is done for ##V## but it is the same reasoning for ##T##, you write the potential, ( or kinetic energy in your case), which is a scalar, and it is the same in the two coordinates systems. It is done in eq. (36) of your file. In case you are worrying, you know that the energy is the same in the two systems of corrdinates, as you are limiting yourself to a specific kind of transofrmations ( it is stated in the text). Then you inject the transformation to express ##q'## in function of ##q##, eq. (37) in text, and indentify to have the relation between ##T'## and ##T##.

I think you are lacking a bit of practice in pure linear algebra, and it is usual to be confused at first when you try to use it in physics, it is not trivial when you are studying it.:smile:

What I would recommend you is to pratice pure linear algebra, only the math side of it, without Physics, especially to train you in changing of coordinates and basis, reduction, diagonalisation and euclidean spaces. You have to master those notions to do this kind of physics in peace.

I'm sorry I cannot cite you good textbooks on the subject, as I didn't learn math in English, but I'm sure you will find plenty of them !
 
  • Like
Likes JD_PM
  • #6
patfinder said:
which corresponds to the transformation ## q'' = S_{2} S_{1} ## and gives ##T'' = I##.
I'm sorry I did a typing mistake, I was trying to write which corresponds to the transformation ## q'' = S_{2} S_{1}q ## ' off course.
 
  • #7
patfinder said:
Writing this should give you a shock ! You are wirting ##T## a matrix, ## = \ \dot{\vec q'}^T T' \dot q '## the kinetic energy, which is a scalar, a number ! So you are starting from a wrong relation.

Ups I was sloppy, thanks for pointing it out. My answers do not change though.

There's a discrepancy between your work and mine which I'll highlight once I rewrite my #3 and #4.

Rewriting my #3 (slightly modified)

If we assume that energies do not change under coordinate transformations we can write

$$\dot{\vec q}^T T \dot{\vec q} = \dot{\vec q'}^T T' \dot{\vec q'}$$

If we apply the coordinate transformation ##\vec q' = S_1 \vec q \ ## to ##\dot{\vec q'}^T T' \dot{\vec q'}## we get

$$\dot{\vec q}^T T \dot{\vec q}=(S_1 \dot{\vec q})^T T'(S_1 \dot{\vec q}) = \dot{\vec q}^T S_1^T T'S_1 \dot{\vec q} \tag1$$

Thus we have

$$T = S_1^T T'S_1 \iff T'=S_1^{-T} T S_1^{-1} \tag2$$

As ##T'## is a real symmetric matrix, it is diagonalizable. Thus we can assert that

$$T' = D \tag3$$

Next if we apply the coordinate transformation ##\vec q'' = S_2 \vec q' \ ## to ##\dot{\vec q''}^T T'' \dot{\vec q''}## we get

$$\dot{\vec q'}^T T' \dot{\vec q'}=(S_2 \dot{\vec q'})^T T''(S_2 \dot{\vec q'}) = \dot{\vec q'}^T S_2^T T''S_2 \dot{\vec q'} \tag4$$

Thus we have

$$T' = S_2^T T''S_2 \iff T''=S_2^{-T} T' S_1^{-1} \tag5$$

If we set ##S_2 = \sqrt{D}## then, out of ##(5)## and using ##(3)##, we get

$$T'' = \Big( \sqrt{D} \Big)^{-T} T'\Big( \sqrt{D} \Big)^{-1}=\Big( \sqrt{D} \Big)^{-T} D\Big( \sqrt{D} \Big)^{-1}=\Big( \sqrt{D} \Big)^{-1} D\Big( \sqrt{D} \Big)^{-1}=1 \tag6$$

Where I have used the fact that the inverse matrix of a non-singular symmetric matrix (in this case ##\Big( \sqrt{D} \Big)^{-1}##) is symmetric (i.e. ##\Big( \sqrt{D} \Big)^{-T}=\Big( \sqrt{D} \Big)^{-1}##). Note I do not assume that ##S_2## is an orthogonal matrix here.

Rewriting my #4

Now I am stuck in two basic aspects:

1) To justify ##(3)## I said 'As ##T'## is a real symmetric matrix, it is diagonalizable'; OK but I'd like to work it out explicitly. My idea is this: let's use ##(2)##

$$T'= S_1^{-T} T S_1^{-1} = (S_1^{T})^{-1} D S_1^{-1} = (S_1^{-1})^{-1} D S_1^{-1} = S_1 D S_1^{-1} \tag7$$

Where I assumed that ##S_1## is an orthogonal matrix.

My issue here is that I do not see why ##S_1 D S_1^{-1}=D##. If we set ##S_1=D## we indeed get ##T'=D## sure. However, I do not think that's the whole story.

2) Something interesting happens if we assume ##S_2## to be an orthogonal matrix.

$$T'' = \Big( \sqrt{D}^{-1} \Big)^{T} T'\Big( \sqrt{D} \Big)^{-1}=\Big( \sqrt{D}^{-1} \Big)^{-1} T'\Big( \sqrt{D} \Big)^{-1}=\sqrt{D} D\Big( \sqrt{D} \Big)^{-1}=D \tag8$$

We get ##T'' = D## instead of ##T'' = 1##.

So making the assumption that ##(\sqrt{D})^{-1}## is orthogonal seems to be incorrect based on my analysis. However, in your proof at #2, you assumed ##S_2## to be so and you ended up getting the expected answer.

Mmm what am I missing?
 

1. What are small oscillations?

Small oscillations refer to the back-and-forth motion of a system around a stable equilibrium point. These oscillations occur when the system is disturbed from its equilibrium position by a small amount and then returns to its original position due to the restoring force.

2. How are small oscillations different from large oscillations?

The main difference between small and large oscillations is the amplitude or magnitude of the oscillations. Small oscillations have a small amplitude and occur around a stable equilibrium point, while large oscillations have a larger amplitude and can occur around an unstable equilibrium point.

3. What are spatial transformations in the context of small oscillations?

Spatial transformations refer to changes in the position or orientation of a system in space. In the context of small oscillations, spatial transformations can affect the behavior of the system and how it oscillates around its equilibrium point.

4. How do spatial transformations affect small oscillations?

Spatial transformations can affect small oscillations by changing the equilibrium point of the system, altering the direction and magnitude of the restoring force, and potentially causing the system to exhibit different types of oscillations (e.g. simple harmonic, damped, or forced oscillations).

5. Why are small oscillations important in scientific research?

Small oscillations are important in scientific research because they are a common phenomenon in many physical systems and can provide valuable insights into the behavior and properties of these systems. They also serve as a basis for more complex oscillatory systems and can be used to model and analyze various real-world phenomena.

Similar threads

  • Classical Physics
Replies
1
Views
592
  • Classical Physics
Replies
4
Views
724
  • Classical Physics
Replies
8
Views
1K
  • Classical Physics
2
Replies
53
Views
3K
Replies
2
Views
568
  • Classical Physics
Replies
6
Views
1K
Replies
2
Views
2K
Replies
3
Views
939
  • Calculus and Beyond Homework Help
Replies
2
Views
156
Replies
8
Views
1K
Back
Top